LSAT and Law School Admissions Forum

Get expert LSAT preparation and law school admissions advice from PowerScore Test Preparation.

 Administrator
PowerScore Staff
  • PowerScore Staff
  • Posts: 8917
  • Joined: Feb 02, 2011
|
#34866
Complete Question Explanation

Flaw in the Reasoning. The correct answer choice is (A)

This editorialist discusses the issue of unauthorized music-sharing services, and the complaint
among musicians that such services steal royalties from the musicians who create the songs. Based
on the fact that many other parties (record companies, music publishers, managers, etc.) take an
unfair cut of the revenues from music sales, however, the editorialist asserts that music-sharing
services are not to blame. The questionable argument can be broken down as follow:

..... Premise: ..... Musicians claim that unauthorized music-sharing services take their
..... ..... ..... ..... royalties.

..... Counter-premise: However, record companies, publishers, managers, and others take more
..... ..... ..... ..... than their fair share of revenues.

..... Conclusion: ..... Thus, unauthorized music-sharing services are not to blame.

The problem with this argument is that the author responds to one accusation by simply shifting
the blame. The fact that many other parties take unfair portions of music revenue does not absolve
music-sharing services of all blame. Since the question stem asks for the flaw in the author's
reasoning, the right answer choice should reference this logical flaw.

Answer choice (A): This is the correct answer choice. As discussed, the author concludes that
music-sharing services are not at fault simply because there are several other parties that also take
inequitable shares of music revenue.

Answer choice (B): The author defends the actions of music-sharing companies, but doesn't go so far
as to promote a particular type of behavior, so this choice should be ruled out of contention.

Answer choice (C): This answer choice describes an ad hominem attack on the character of a person,
but since there is no such attack in the stimulus, this choice cannot be the right answer to this Flaw
question.

Answer choice (D): The author does not point to any undesirable consequence of thinking that
music-sharing services rob musicians of royalties; this is not how the author tries to show that the
position is false, so this choice cannot be correct.

Answer choice (E): This choice describes the conditional reasoning flaw of confusing a necessary
condition for a sufficient condition, but as discussed, this is not the logical flaw that is reflected in
the author's argument.
 Leela
  • Posts: 63
  • Joined: Apr 13, 2019
|
#64503
I understand that the editorialist is shifting the blame off of the music-sharing services, but could the stimulus not be read in such a way that the editorialist believes no one is to blame? I prephrased that because everyone (music-sharing services, record companies, publishers, managers, and other intermediaries) robs the musicians, one group not being to blame doesn't make sense.
 Rachael Wilkenfeld
PowerScore Staff
  • PowerScore Staff
  • Posts: 1358
  • Joined: Dec 15, 2011
|
#64522
Hi Leela,

That's an interesting read of the stimulus, but I don't think we can support that the author thinks NO ONE is to blame. His conclusion is just that we can't blame the streaming services, because, look, these other players also cut into the musicians' revenue.

Ultimately, it doesn't matter if the author thinks no one is to blame because there are too many bad actors, or just that we can't blame the streaming services. The author is trying to shift blame away from a party not because their actions aren't blameworthy, but just because someone else has committed bad acts. That's flawed reasoning. Other people's bad actions have no impact on the goodness/badness of anyone else's actions. The music companies' actions here are just a distraction from the issue of the streaming services' actions.

Hope that helps!
Rachael
User avatar
 chrisfromc123
  • Posts: 5
  • Joined: May 08, 2021
|
#86928
I think I might be confused about necessary vs sufficient condition, but couldn't this qualify as answer E?

For example, if it was necessary that someone 1. take a cut of their royalties and 2. do so illegally, would this not then lead to them being sufficiently blameworthy of "robbing" someone of their royalties?.

As a result, while companies, publishers and managers take a sizeable cut of royalties, they are doing so in a contractually based relationship.

On the other hand, unauthorized moving streaming websites do so illicitly.

I can see why answer A can be correct. However, it seemed to me like the basis of placing blame on the unauthorized streamers was not being addressed through their argument as they both fulfilled necessary conditions 1 and 2 while the publishers and managers only fulfilled necessary condition 1 (and thereby did not fulfill the sufficient condition of royalties and illegal).

Thanks
User avatar
 Poonam Agrawal
PowerScore Staff
  • PowerScore Staff
  • Posts: 71
  • Joined: Apr 23, 2021
|
#86991
Hi Chris!

The argument does not introduce any "necessary conditions for blameworthiness." If it did, the stimulus would say something along the lines of, it is necessary for a company to do X for it to be assigned blame. Without any conditional reasoning (the usage of if-then logic) present in the stimulus, you cannot justify picking answer choice (E).

Answer choice (A) is the correct answer here because it identifies the flaw of concluding that music-sharing services are not to be blamed just because another party is also doing unfair things. Consider this parallel argument: You have just written a book. I'm making copies of your book and giving them away for free, denying you of the revenue that you would have made from your sales. However, you can't blame me because your book publisher is also taking your money. See how that's a flawed argument? Just because one party is doing unfair things, it does not mean that the other party is also not to be blamed.

This is why answer choice (A) is the best answer to this question. Answer choice (E) might have worked if the argument itself introduced the necessary conditions that you used in your example, but as written, it does not. Hope this helps!

Get the most out of your LSAT Prep Plus subscription.

Analyze and track your performance with our Testing and Analytics Package.